Proving that a differentiable function f, with f' bounded, is uniformly continuous












0












$begingroup$



Let $f: R rightarrow R$ be a differentiable function. Prove that if $f'$ is bounded, then $f$ is uniformly continuous




My attempt:



Since $f$ is differentiable, we have $f'(x) = lim_{x to x_0}frac{f(x) - f(x_0)}{x-x_0}$.



Since $f$ is differentiable, it follows that $f$ is also continuous. Therefore, given $epsilon > 0$, there is some $delta > 0$ such that $|x-x_0|<delta rightarrow |f(x)-f(x_0)|<epsilon$.



$frac{|x-x_0|}{|x-x_0|}*|f(x)-f(x_0)|=|x-x_0|frac{|f(x)-f(x_0)|}{|x-x_0|}<deltafrac{|f(x)-f(x_0)|}{|x-x_0|}<epsilon$



Now take $lim_{x to x_0}$ of both sides to get: $delta f'(x) < epsilon$. Since $f'$ is bounded, there is some $M$ such that $-M leq f' leq M$. So we have $delta f'(x)leqdelta M<epsilon$. Take $delta = frac{epsilon}{M}$. Therefore, $f$ is uniformly continuous.



Is my logic here correct? I am not sure if I can do the limit step.










share|cite|improve this question











$endgroup$












  • $begingroup$
    I find your “proof” hard to follow and check. You should rewrite it while defining precisely all your variables and checking dependencies.
    $endgroup$
    – Mindlack
    Dec 12 '18 at 22:45






  • 2




    $begingroup$
    Yes. Use Lagrange's Mean Value theorem. $|f(x)-f(x_0)|=|f'(c)(x-x_0)|leq M|x-x_0|$ where $cin(x,x_0)$ and $Minmathbb{R}:::|f'(x)|leq M:forall :xinmathbb{R}$. Infact it is $M$-Lipschitz.
    $endgroup$
    – Yadati Kiran
    Dec 12 '18 at 22:48
















0












$begingroup$



Let $f: R rightarrow R$ be a differentiable function. Prove that if $f'$ is bounded, then $f$ is uniformly continuous




My attempt:



Since $f$ is differentiable, we have $f'(x) = lim_{x to x_0}frac{f(x) - f(x_0)}{x-x_0}$.



Since $f$ is differentiable, it follows that $f$ is also continuous. Therefore, given $epsilon > 0$, there is some $delta > 0$ such that $|x-x_0|<delta rightarrow |f(x)-f(x_0)|<epsilon$.



$frac{|x-x_0|}{|x-x_0|}*|f(x)-f(x_0)|=|x-x_0|frac{|f(x)-f(x_0)|}{|x-x_0|}<deltafrac{|f(x)-f(x_0)|}{|x-x_0|}<epsilon$



Now take $lim_{x to x_0}$ of both sides to get: $delta f'(x) < epsilon$. Since $f'$ is bounded, there is some $M$ such that $-M leq f' leq M$. So we have $delta f'(x)leqdelta M<epsilon$. Take $delta = frac{epsilon}{M}$. Therefore, $f$ is uniformly continuous.



Is my logic here correct? I am not sure if I can do the limit step.










share|cite|improve this question











$endgroup$












  • $begingroup$
    I find your “proof” hard to follow and check. You should rewrite it while defining precisely all your variables and checking dependencies.
    $endgroup$
    – Mindlack
    Dec 12 '18 at 22:45






  • 2




    $begingroup$
    Yes. Use Lagrange's Mean Value theorem. $|f(x)-f(x_0)|=|f'(c)(x-x_0)|leq M|x-x_0|$ where $cin(x,x_0)$ and $Minmathbb{R}:::|f'(x)|leq M:forall :xinmathbb{R}$. Infact it is $M$-Lipschitz.
    $endgroup$
    – Yadati Kiran
    Dec 12 '18 at 22:48














0












0








0





$begingroup$



Let $f: R rightarrow R$ be a differentiable function. Prove that if $f'$ is bounded, then $f$ is uniformly continuous




My attempt:



Since $f$ is differentiable, we have $f'(x) = lim_{x to x_0}frac{f(x) - f(x_0)}{x-x_0}$.



Since $f$ is differentiable, it follows that $f$ is also continuous. Therefore, given $epsilon > 0$, there is some $delta > 0$ such that $|x-x_0|<delta rightarrow |f(x)-f(x_0)|<epsilon$.



$frac{|x-x_0|}{|x-x_0|}*|f(x)-f(x_0)|=|x-x_0|frac{|f(x)-f(x_0)|}{|x-x_0|}<deltafrac{|f(x)-f(x_0)|}{|x-x_0|}<epsilon$



Now take $lim_{x to x_0}$ of both sides to get: $delta f'(x) < epsilon$. Since $f'$ is bounded, there is some $M$ such that $-M leq f' leq M$. So we have $delta f'(x)leqdelta M<epsilon$. Take $delta = frac{epsilon}{M}$. Therefore, $f$ is uniformly continuous.



Is my logic here correct? I am not sure if I can do the limit step.










share|cite|improve this question











$endgroup$





Let $f: R rightarrow R$ be a differentiable function. Prove that if $f'$ is bounded, then $f$ is uniformly continuous




My attempt:



Since $f$ is differentiable, we have $f'(x) = lim_{x to x_0}frac{f(x) - f(x_0)}{x-x_0}$.



Since $f$ is differentiable, it follows that $f$ is also continuous. Therefore, given $epsilon > 0$, there is some $delta > 0$ such that $|x-x_0|<delta rightarrow |f(x)-f(x_0)|<epsilon$.



$frac{|x-x_0|}{|x-x_0|}*|f(x)-f(x_0)|=|x-x_0|frac{|f(x)-f(x_0)|}{|x-x_0|}<deltafrac{|f(x)-f(x_0)|}{|x-x_0|}<epsilon$



Now take $lim_{x to x_0}$ of both sides to get: $delta f'(x) < epsilon$. Since $f'$ is bounded, there is some $M$ such that $-M leq f' leq M$. So we have $delta f'(x)leqdelta M<epsilon$. Take $delta = frac{epsilon}{M}$. Therefore, $f$ is uniformly continuous.



Is my logic here correct? I am not sure if I can do the limit step.







real-analysis proof-verification uniform-continuity






share|cite|improve this question















share|cite|improve this question













share|cite|improve this question




share|cite|improve this question








edited Dec 12 '18 at 22:49









Theo Bendit

20.2k12353




20.2k12353










asked Dec 12 '18 at 22:41









hkj447hkj447

636




636












  • $begingroup$
    I find your “proof” hard to follow and check. You should rewrite it while defining precisely all your variables and checking dependencies.
    $endgroup$
    – Mindlack
    Dec 12 '18 at 22:45






  • 2




    $begingroup$
    Yes. Use Lagrange's Mean Value theorem. $|f(x)-f(x_0)|=|f'(c)(x-x_0)|leq M|x-x_0|$ where $cin(x,x_0)$ and $Minmathbb{R}:::|f'(x)|leq M:forall :xinmathbb{R}$. Infact it is $M$-Lipschitz.
    $endgroup$
    – Yadati Kiran
    Dec 12 '18 at 22:48


















  • $begingroup$
    I find your “proof” hard to follow and check. You should rewrite it while defining precisely all your variables and checking dependencies.
    $endgroup$
    – Mindlack
    Dec 12 '18 at 22:45






  • 2




    $begingroup$
    Yes. Use Lagrange's Mean Value theorem. $|f(x)-f(x_0)|=|f'(c)(x-x_0)|leq M|x-x_0|$ where $cin(x,x_0)$ and $Minmathbb{R}:::|f'(x)|leq M:forall :xinmathbb{R}$. Infact it is $M$-Lipschitz.
    $endgroup$
    – Yadati Kiran
    Dec 12 '18 at 22:48
















$begingroup$
I find your “proof” hard to follow and check. You should rewrite it while defining precisely all your variables and checking dependencies.
$endgroup$
– Mindlack
Dec 12 '18 at 22:45




$begingroup$
I find your “proof” hard to follow and check. You should rewrite it while defining precisely all your variables and checking dependencies.
$endgroup$
– Mindlack
Dec 12 '18 at 22:45




2




2




$begingroup$
Yes. Use Lagrange's Mean Value theorem. $|f(x)-f(x_0)|=|f'(c)(x-x_0)|leq M|x-x_0|$ where $cin(x,x_0)$ and $Minmathbb{R}:::|f'(x)|leq M:forall :xinmathbb{R}$. Infact it is $M$-Lipschitz.
$endgroup$
– Yadati Kiran
Dec 12 '18 at 22:48




$begingroup$
Yes. Use Lagrange's Mean Value theorem. $|f(x)-f(x_0)|=|f'(c)(x-x_0)|leq M|x-x_0|$ where $cin(x,x_0)$ and $Minmathbb{R}:::|f'(x)|leq M:forall :xinmathbb{R}$. Infact it is $M$-Lipschitz.
$endgroup$
– Yadati Kiran
Dec 12 '18 at 22:48










1 Answer
1






active

oldest

votes


















1












$begingroup$

The $delta$ you end up choosing is good; this $delta$ will work. The problem is your justification why it works. It almost works with a bit of unpacking, but it needs to be rewritten in a way that is clear flows logically from the assumption that $|x - x_0| < delta = varepsilon/M$ to the conclusion that $|f(x) - f(x_0)| < varepsilon$.



You write




Since $f$ is differentiable, it follows that $f$ is also continuous. Therefore, given $varepsilon > 0$, there is some $delta > 0$ such that $|x-x_0|<delta rightarrow |f(x)-f(x_0)|<varepsilon$.




This is true, but it's a confusing step to put in your proof. The way I interepeted this when I first read it is that you are choosing a value for the variable $delta$. You don't know what it is, but you know one exists that satisfies the above property (given the fixed $varepsilon > 0$ and $x_0 in mathbb{R}$). The rest of your proof would then (somehow) have to justify why this $delta$ does not depend on $x_0$ at all, or perhaps define another $delta'$, based on $delta$, that didn't depend on $x_0$. Since you end up simply defining a totally different $delta$, not relating to this $delta$, the step is confusing, and probably should be dropped.



Next, you write




$frac{|x-x_0|}{|x-x_0|} times |f(x)-f(x_0)|=|x-x_0|frac{|f(x)-f(x_0)|}{|x-x_0|}<deltafrac{|f(x)-f(x_0)|}{|x-x_0|}<varepsilon$




This is another reason why the previous step is confusing; you are now using $delta$ in inequalities. Without first defining $delta$, this is a bit confusing. You haven't talked about what you're assuming here. For example, it appears that you're assuming $|x - x_0| < delta$. If you have a $delta$ defined (such as from the previous paragraph), this is a very reasonable assumption to make, but it should be made explicit before writing this step.



It's also not clear why $deltafrac{|f(x)-f(x_0)|}{|x-x_0|}<varepsilon$. Without a clear definition of $delta$, I don't know why this would be true.



You also write




Now take $lim_{x to x_0}$ of both sides to get: $delta f'(x) < epsilon$.




This is a small point: if you have $g(x) < N$ for all $x$, then $lim_{x to a} g(x) le N$. You cannot conclude strict inequality!






share|cite|improve this answer









$endgroup$














    Your Answer





    StackExchange.ifUsing("editor", function () {
    return StackExchange.using("mathjaxEditing", function () {
    StackExchange.MarkdownEditor.creationCallbacks.add(function (editor, postfix) {
    StackExchange.mathjaxEditing.prepareWmdForMathJax(editor, postfix, [["$", "$"], ["\\(","\\)"]]);
    });
    });
    }, "mathjax-editing");

    StackExchange.ready(function() {
    var channelOptions = {
    tags: "".split(" "),
    id: "69"
    };
    initTagRenderer("".split(" "), "".split(" "), channelOptions);

    StackExchange.using("externalEditor", function() {
    // Have to fire editor after snippets, if snippets enabled
    if (StackExchange.settings.snippets.snippetsEnabled) {
    StackExchange.using("snippets", function() {
    createEditor();
    });
    }
    else {
    createEditor();
    }
    });

    function createEditor() {
    StackExchange.prepareEditor({
    heartbeatType: 'answer',
    autoActivateHeartbeat: false,
    convertImagesToLinks: true,
    noModals: true,
    showLowRepImageUploadWarning: true,
    reputationToPostImages: 10,
    bindNavPrevention: true,
    postfix: "",
    imageUploader: {
    brandingHtml: "Powered by u003ca class="icon-imgur-white" href="https://imgur.com/"u003eu003c/au003e",
    contentPolicyHtml: "User contributions licensed under u003ca href="https://creativecommons.org/licenses/by-sa/3.0/"u003ecc by-sa 3.0 with attribution requiredu003c/au003e u003ca href="https://stackoverflow.com/legal/content-policy"u003e(content policy)u003c/au003e",
    allowUrls: true
    },
    noCode: true, onDemand: true,
    discardSelector: ".discard-answer"
    ,immediatelyShowMarkdownHelp:true
    });


    }
    });














    draft saved

    draft discarded


















    StackExchange.ready(
    function () {
    StackExchange.openid.initPostLogin('.new-post-login', 'https%3a%2f%2fmath.stackexchange.com%2fquestions%2f3037327%2fproving-that-a-differentiable-function-f-with-f-bounded-is-uniformly-continuo%23new-answer', 'question_page');
    }
    );

    Post as a guest















    Required, but never shown

























    1 Answer
    1






    active

    oldest

    votes








    1 Answer
    1






    active

    oldest

    votes









    active

    oldest

    votes






    active

    oldest

    votes









    1












    $begingroup$

    The $delta$ you end up choosing is good; this $delta$ will work. The problem is your justification why it works. It almost works with a bit of unpacking, but it needs to be rewritten in a way that is clear flows logically from the assumption that $|x - x_0| < delta = varepsilon/M$ to the conclusion that $|f(x) - f(x_0)| < varepsilon$.



    You write




    Since $f$ is differentiable, it follows that $f$ is also continuous. Therefore, given $varepsilon > 0$, there is some $delta > 0$ such that $|x-x_0|<delta rightarrow |f(x)-f(x_0)|<varepsilon$.




    This is true, but it's a confusing step to put in your proof. The way I interepeted this when I first read it is that you are choosing a value for the variable $delta$. You don't know what it is, but you know one exists that satisfies the above property (given the fixed $varepsilon > 0$ and $x_0 in mathbb{R}$). The rest of your proof would then (somehow) have to justify why this $delta$ does not depend on $x_0$ at all, or perhaps define another $delta'$, based on $delta$, that didn't depend on $x_0$. Since you end up simply defining a totally different $delta$, not relating to this $delta$, the step is confusing, and probably should be dropped.



    Next, you write




    $frac{|x-x_0|}{|x-x_0|} times |f(x)-f(x_0)|=|x-x_0|frac{|f(x)-f(x_0)|}{|x-x_0|}<deltafrac{|f(x)-f(x_0)|}{|x-x_0|}<varepsilon$




    This is another reason why the previous step is confusing; you are now using $delta$ in inequalities. Without first defining $delta$, this is a bit confusing. You haven't talked about what you're assuming here. For example, it appears that you're assuming $|x - x_0| < delta$. If you have a $delta$ defined (such as from the previous paragraph), this is a very reasonable assumption to make, but it should be made explicit before writing this step.



    It's also not clear why $deltafrac{|f(x)-f(x_0)|}{|x-x_0|}<varepsilon$. Without a clear definition of $delta$, I don't know why this would be true.



    You also write




    Now take $lim_{x to x_0}$ of both sides to get: $delta f'(x) < epsilon$.




    This is a small point: if you have $g(x) < N$ for all $x$, then $lim_{x to a} g(x) le N$. You cannot conclude strict inequality!






    share|cite|improve this answer









    $endgroup$


















      1












      $begingroup$

      The $delta$ you end up choosing is good; this $delta$ will work. The problem is your justification why it works. It almost works with a bit of unpacking, but it needs to be rewritten in a way that is clear flows logically from the assumption that $|x - x_0| < delta = varepsilon/M$ to the conclusion that $|f(x) - f(x_0)| < varepsilon$.



      You write




      Since $f$ is differentiable, it follows that $f$ is also continuous. Therefore, given $varepsilon > 0$, there is some $delta > 0$ such that $|x-x_0|<delta rightarrow |f(x)-f(x_0)|<varepsilon$.




      This is true, but it's a confusing step to put in your proof. The way I interepeted this when I first read it is that you are choosing a value for the variable $delta$. You don't know what it is, but you know one exists that satisfies the above property (given the fixed $varepsilon > 0$ and $x_0 in mathbb{R}$). The rest of your proof would then (somehow) have to justify why this $delta$ does not depend on $x_0$ at all, or perhaps define another $delta'$, based on $delta$, that didn't depend on $x_0$. Since you end up simply defining a totally different $delta$, not relating to this $delta$, the step is confusing, and probably should be dropped.



      Next, you write




      $frac{|x-x_0|}{|x-x_0|} times |f(x)-f(x_0)|=|x-x_0|frac{|f(x)-f(x_0)|}{|x-x_0|}<deltafrac{|f(x)-f(x_0)|}{|x-x_0|}<varepsilon$




      This is another reason why the previous step is confusing; you are now using $delta$ in inequalities. Without first defining $delta$, this is a bit confusing. You haven't talked about what you're assuming here. For example, it appears that you're assuming $|x - x_0| < delta$. If you have a $delta$ defined (such as from the previous paragraph), this is a very reasonable assumption to make, but it should be made explicit before writing this step.



      It's also not clear why $deltafrac{|f(x)-f(x_0)|}{|x-x_0|}<varepsilon$. Without a clear definition of $delta$, I don't know why this would be true.



      You also write




      Now take $lim_{x to x_0}$ of both sides to get: $delta f'(x) < epsilon$.




      This is a small point: if you have $g(x) < N$ for all $x$, then $lim_{x to a} g(x) le N$. You cannot conclude strict inequality!






      share|cite|improve this answer









      $endgroup$
















        1












        1








        1





        $begingroup$

        The $delta$ you end up choosing is good; this $delta$ will work. The problem is your justification why it works. It almost works with a bit of unpacking, but it needs to be rewritten in a way that is clear flows logically from the assumption that $|x - x_0| < delta = varepsilon/M$ to the conclusion that $|f(x) - f(x_0)| < varepsilon$.



        You write




        Since $f$ is differentiable, it follows that $f$ is also continuous. Therefore, given $varepsilon > 0$, there is some $delta > 0$ such that $|x-x_0|<delta rightarrow |f(x)-f(x_0)|<varepsilon$.




        This is true, but it's a confusing step to put in your proof. The way I interepeted this when I first read it is that you are choosing a value for the variable $delta$. You don't know what it is, but you know one exists that satisfies the above property (given the fixed $varepsilon > 0$ and $x_0 in mathbb{R}$). The rest of your proof would then (somehow) have to justify why this $delta$ does not depend on $x_0$ at all, or perhaps define another $delta'$, based on $delta$, that didn't depend on $x_0$. Since you end up simply defining a totally different $delta$, not relating to this $delta$, the step is confusing, and probably should be dropped.



        Next, you write




        $frac{|x-x_0|}{|x-x_0|} times |f(x)-f(x_0)|=|x-x_0|frac{|f(x)-f(x_0)|}{|x-x_0|}<deltafrac{|f(x)-f(x_0)|}{|x-x_0|}<varepsilon$




        This is another reason why the previous step is confusing; you are now using $delta$ in inequalities. Without first defining $delta$, this is a bit confusing. You haven't talked about what you're assuming here. For example, it appears that you're assuming $|x - x_0| < delta$. If you have a $delta$ defined (such as from the previous paragraph), this is a very reasonable assumption to make, but it should be made explicit before writing this step.



        It's also not clear why $deltafrac{|f(x)-f(x_0)|}{|x-x_0|}<varepsilon$. Without a clear definition of $delta$, I don't know why this would be true.



        You also write




        Now take $lim_{x to x_0}$ of both sides to get: $delta f'(x) < epsilon$.




        This is a small point: if you have $g(x) < N$ for all $x$, then $lim_{x to a} g(x) le N$. You cannot conclude strict inequality!






        share|cite|improve this answer









        $endgroup$



        The $delta$ you end up choosing is good; this $delta$ will work. The problem is your justification why it works. It almost works with a bit of unpacking, but it needs to be rewritten in a way that is clear flows logically from the assumption that $|x - x_0| < delta = varepsilon/M$ to the conclusion that $|f(x) - f(x_0)| < varepsilon$.



        You write




        Since $f$ is differentiable, it follows that $f$ is also continuous. Therefore, given $varepsilon > 0$, there is some $delta > 0$ such that $|x-x_0|<delta rightarrow |f(x)-f(x_0)|<varepsilon$.




        This is true, but it's a confusing step to put in your proof. The way I interepeted this when I first read it is that you are choosing a value for the variable $delta$. You don't know what it is, but you know one exists that satisfies the above property (given the fixed $varepsilon > 0$ and $x_0 in mathbb{R}$). The rest of your proof would then (somehow) have to justify why this $delta$ does not depend on $x_0$ at all, or perhaps define another $delta'$, based on $delta$, that didn't depend on $x_0$. Since you end up simply defining a totally different $delta$, not relating to this $delta$, the step is confusing, and probably should be dropped.



        Next, you write




        $frac{|x-x_0|}{|x-x_0|} times |f(x)-f(x_0)|=|x-x_0|frac{|f(x)-f(x_0)|}{|x-x_0|}<deltafrac{|f(x)-f(x_0)|}{|x-x_0|}<varepsilon$




        This is another reason why the previous step is confusing; you are now using $delta$ in inequalities. Without first defining $delta$, this is a bit confusing. You haven't talked about what you're assuming here. For example, it appears that you're assuming $|x - x_0| < delta$. If you have a $delta$ defined (such as from the previous paragraph), this is a very reasonable assumption to make, but it should be made explicit before writing this step.



        It's also not clear why $deltafrac{|f(x)-f(x_0)|}{|x-x_0|}<varepsilon$. Without a clear definition of $delta$, I don't know why this would be true.



        You also write




        Now take $lim_{x to x_0}$ of both sides to get: $delta f'(x) < epsilon$.




        This is a small point: if you have $g(x) < N$ for all $x$, then $lim_{x to a} g(x) le N$. You cannot conclude strict inequality!







        share|cite|improve this answer












        share|cite|improve this answer



        share|cite|improve this answer










        answered Dec 12 '18 at 23:17









        Theo BenditTheo Bendit

        20.2k12353




        20.2k12353






























            draft saved

            draft discarded




















































            Thanks for contributing an answer to Mathematics Stack Exchange!


            • Please be sure to answer the question. Provide details and share your research!

            But avoid



            • Asking for help, clarification, or responding to other answers.

            • Making statements based on opinion; back them up with references or personal experience.


            Use MathJax to format equations. MathJax reference.


            To learn more, see our tips on writing great answers.




            draft saved


            draft discarded














            StackExchange.ready(
            function () {
            StackExchange.openid.initPostLogin('.new-post-login', 'https%3a%2f%2fmath.stackexchange.com%2fquestions%2f3037327%2fproving-that-a-differentiable-function-f-with-f-bounded-is-uniformly-continuo%23new-answer', 'question_page');
            }
            );

            Post as a guest















            Required, but never shown





















































            Required, but never shown














            Required, but never shown












            Required, but never shown







            Required, but never shown

































            Required, but never shown














            Required, but never shown












            Required, but never shown







            Required, but never shown







            Popular posts from this blog

            Biblatex bibliography style without URLs when DOI exists (in Overleaf with Zotero bibliography)

            ComboBox Display Member on multiple fields

            Is it possible to collect Nectar points via Trainline?